1answer.
Ask question
Login Signup
Ask question
All categories
  • English
  • Mathematics
  • Social Studies
  • Business
  • History
  • Health
  • Geography
  • Biology
  • Physics
  • Chemistry
  • Computers and Technology
  • Arts
  • World Languages
  • Spanish
  • French
  • German
  • Advanced Placement (AP)
  • SAT
  • Medicine
  • Law
  • Engineering
Bad White [126]
3 years ago
12

What is the greatest value of 2,463.9051

Mathematics
1 answer:
RoseWind [281]3 years ago
6 0

Hey There @Bre18016,

The answer is \boxed{2}

The greatest value of 2,463.9051 would be the thousands place (2) simply as it is the biggest number out of the other places.

For instance, if we had the number 300, 3 would be the greatest value.

Or let's say we had 10,000 the 1 would be the greatest value.

Furthermore, you could look at the first digit in the entire number to deter mine the greatest value.

You might be interested in
The diagram represents 6x^2-7x+2with the factor of 2x-1. What is the factor of 6 x^2-7x+2
Maksim231197 [3]

Answer:

(6x²-7x+2)/2x-1

(2x-1)(3x-2)/2x-1

=

<h2>3x-2</h2>
5 0
3 years ago
Read 2 more answers
14. Solve the equation above for h​
Westkost [7]

Answer:

h=69

Step-by-step explanation:

H=VM2 +69=Y.......

5 0
4 years ago
Use the identity (x+y)(x^2−xy+y^2)=x^3+y^3 to find the sum of two numbers if the product of the numbers is 28, the sum of the sq
lara31 [8.8K]

Step-by-step explanation:

We have xy = 28, x² + y² = 65 and x³ + y³ = 407.

Since (x + y)(x² - xy + y²) = x³ + y³,

x + y = (x³ + y³)/(x² + y² - xy)

= (407) / [(65) - (28)]

= 407 / 37

= 11.

Hence the sum of the numbers is 11.

4 0
3 years ago
Read 2 more answers
CAN SOMEONE PLEASE HELP
Sunny_sXe [5.5K]

It is most probably 30

cause you add 24, 30, 36

then divide it by 3

that is how i learnt it

5 0
3 years ago
Subtract 275000-30000
Varvara68 [4.7K]
Hi, it is 245,000 all you have to do is 70000-3000
5 0
3 years ago
Read 2 more answers
Other questions:
  • What is the ratio of people who prefer oranges to bananas?
    12·1 answer
  • E=-2.4t+75 how far does the anchor drop every 5 seconds?
    11·1 answer
  • If it takes 1074 digits to number the pages of a book starting with page 1. how many pages are in the book? The book has a total
    5·1 answer
  • What is side e<br><br>Angle d and angle f<br>Please let me know asap
    11·1 answer
  • Which Quaker belief do you think influenced William Penn’s attitude toward Native Americans? Explain your answer.
    14·1 answer
  • There are 16 female performers in a dance recital. The ratio of men to women is 3:4. How many men are in
    7·1 answer
  • Find (4.54×10^8)-(3.98×10^8)
    11·1 answer
  • Don’t really know what linear functions are
    13·1 answer
  • Where do we put the addend Ill give 30 you dont have to answer the sum
    14·2 answers
  • Please help me with this problem.
    5·1 answer
Add answer
Login
Not registered? Fast signup
Signup
Login Signup
Ask question!